Bryant - Course 6. Estate Planning. 4. Gross Estate Flashcards

1
Q

Module Introduction

One of the most tragic events of the 20th century was the unexpected death, at age 36, of Princess Diana on August 31, 1997. After her turbulent divorce, Diana was killed as she and her companion, Dodi Fayed, raced through Paris while being pursued by photographers on motorcycles.
Princess Diana left behind a fortune of approximately $35 million. Her gross estate was composed of stock, jewelry, dresses, and cash, including her $28 million divorce settlement. Most of her fortune was bequeathed to her sons, Prince William and Prince Harry. They will eventually inherit about $11.5 million each, with the inheritance held in trust until that time. However, before they could receive anything, approximately $13.9 million was paid in tax.

Taxes can take a big portion out of large estates, sometimes up to more than half the principal. The good news is that taxes need not take as large a portion of the estate if a financial planner helps the person to value his or her estate properly and take full advantage of the deductions, credits, and estate-planning tools available.

A

Upon completion of this module you should be able to:
* Define the types of assets included in the gross estate
* Explain how interests in property and life estates affect the gross estate
* State the treatment of jointly-owned property and the treatment of assets owned jointly between spouses and non-spouses
* Understand the importance of powers of appointment
* Explain the significance of valuation planning
* Describe the methods used in valuation of different types of property and interests

How well did you know this?
1
Not at all
2
3
4
5
Perfectly
2
Q

Module Overview

The value of the gross estate includes property distributed by the decedent’s will, assets owned & distributed according to the provisions of a living trust, as well as life insurance & retirement accounts. Stocks, bonds, tangible personal property, real property, mortgages, notes, and lifetime transfers that are revocable, or in which the decedent retained an interest, are all included in the decedent’s gross estate.
* Also included in the gross estate is property over which the decedent has a general power of appointment, as well as some lifetime transfers made within three years of the decedent’s death.

A

To ensure that you have a comprehensive understanding of what is included in the gross estate, the following lessons will be covered in this module:
* Gross Estate Assets
* Other IRC Sections
* Valuation Planning

How well did you know this?
1
Not at all
2
3
4
5
Perfectly
3
Q

Section 1 - Gross Estate Assets

The value of the gross estate is not just the total of all property owned by the decedent - it includes the total of all property in which the decedent possessed an interest at the time of death. The value of certain property may be included in the decedent’s gross estate even though someone else holds the legal title to that property. The Internal Revenue Code (IRC) lists the types of assets whose value is to be included in the decedent’s gross estate, as well as the basis on which certain assets may be excluded.

To ensure that you have a solid understanding of what assets are included in the gross estate, the following topics will be covered in this lesson:
* Property Owned Outright (IRC 2033)
* Jointly Owned Property (IRC 2040)
* Dower and Curtesy Interest (IRC 2034)
* Certain Property Transferred (IRC 2035)
* Retained Life Estate (IRC 2036)
* Transfer at Death (IRC 2037)

A

Upon completion of this lesson, you should be able to:
* List the types of property owned outright by the decedent
* Determine the amount of property included in a joint owner’s estate
* Explain the dower and curtesy interest
* Specify the treatment of property transferred within three years of death
* Identify property in which the decedent has retained life estate
* Describe the handling of lifetime transfers
* Define a reversionary interest

How well did you know this?
1
Not at all
2
3
4
5
Perfectly
4
Q

Practitioner Advice:

Practitioner Advice: When the topic of estate planning is initiated during the financial planning process, most clients respond with, “I do not own enough assets to worry about estate planning,” or, “My estate is too small to worry about estate planning.”
* In order to counter this natural resistance to the estate planning process, it is extremely important for the financial planner to have a working understanding of how to determine the value of the client’s estate.
* When they factor in the value of the death benefit proceeds of life insurance policies, retirement plan assets, real estate, and stocks and bonds, many clients will begin to realize that their estate is quite large, requiring the tax planning associated with the estate planning process.

A
How well did you know this?
1
Not at all
2
3
4
5
Perfectly
5
Q

Describe Property Owned Outright (IRC 2033)

A

Internal Revenue Code Section 2033 is a catchall provision dealing with assets, which are included in the gross estate.
* Effectively, IRC Section 2033 states that the value of all property in which the decedent had an interest on the date of death will be included in the decedent’s gross estate, unless a specific exclusion for the property exists.
* Therefore, only the value of the property interest held by the decedent on the date of death will be included in the gross estate.

Type of property interest:

State law determines the character of a property interest. Therefore, if there is any question as to whether an interest owned by the decedent is property under 2033, state law, not the federal IRC, will deal with this question. In any event, examples of property whose value will be included under 2033 include:
Specific property
* stocks
* bonds,
* real estate, etc.
* Property in which the decedent held a sufficient interest.
* Vested remainder interest, as opposed to contingent remainder interest.

How well did you know this?
1
Not at all
2
3
4
5
Perfectly
6
Q

For example, A is trustee of a trust funded by Z with $1 million of assets. The trust is created for the benefit of B, D, and E.
* If A dies, what value of the trust assets will be included in A’s estate?

A

For example, A is trustee of a trust funded by Z with $1 million of assets. The trust is created for the benefit of B, D, and E.
* If A dies, what value of the trust assets will be included in A’s estate?
* Since A was merely the trustee over the assets of the trust, no portion of the trust assets would be included in A’s estate.
* In other words, A was not owner of the assets in his individual capacity, but in a fiduciary capacity for the benefit of B, D, and E.

How well did you know this?
1
Not at all
2
3
4
5
Perfectly
7
Q

For example, If the plan requires A to reach age 65 before he becomes vested, the plan __ ____?? will / will not____ __ be included in his gross estate if he dies at age 60.
* However, if A was already vested in the plan (say it vested at age 55), then the plan __ ____?? would / would not____ __ be included in his gross estate if he dies at age 60.

A

For example, If the plan requires A to reach age 65 before he becomes vested, the plan will not be included in his gross estate if he dies at age 60. This is because A’s interest in the retirement plan is contingent upon his attaining age 65.
* However, if A was already vested in the plan (say it vested at age 55), then the plan would be included in his gross estate if he dies at age 60.

How well did you know this?
1
Not at all
2
3
4
5
Perfectly
8
Q

Describe the Types of Property Included

A

All types of property owned by a decedent outright at death are includable in the gross estate. This includes real and personal property, both tangible and intangible. Intangible personal property such as stocks, bonds, mortgages, notes and other amounts payable to the decedent are includable in the gross estate, as well as tangible personal property such as a decedent’s jewelry and other personal effects.
* However, the decedent must possess more than the bare legal title to property before it can be includable in the estate.

Under state law, if the decedent was the trustee of property or was a strawman owner and had no beneficial interest in the property, no part of such property would be includable in the estate.
* Furthermore, the decedent’s gross estate will include the value of his or her share of certain property held in concert with others.
* For instance, if an individual holds property as a tenant in common with another person, the decedent’s share will be includable as property owned at death.
* Similarly, the value of the decedent’s share of community property will be included in his or her estate.

It is important to note that, no inclusion is required for property in which the decedent’s interest was obtained from someone else and was limited to lifetime enjoyment. This means that an interest that terminated at the decedent’s death and that the decedent had no right to transmit at death will not be included.
* This is known as having a terminable interest in property.
* A husband who gives his wife a terminable interest in property will not receive a gift tax or estate tax marital deduction, and the terminable interest will not be included in the wife’s estate.
* An exception would be Qualified Terminable Interest Property (QTIP).

How well did you know this?
1
Not at all
2
3
4
5
Perfectly
9
Q

Example #1 (Property Inclusion)

Brett gives Eric the right to live in Brett’s home in Miami for as long as Eric lives and nothing more, the value of that home will not be includable in Eric’s estate.
* The value of the home will be included in __ ____??____ __’s estate at death.

A
  • The value of the home will be included in Brett’s estate at death.
How well did you know this?
1
Not at all
2
3
4
5
Perfectly
10
Q

Example #2 (Property Inclusion)

Brett gives Eric’s wife, Pat, the house in Miami for her lifetime and the remainder to Eric, and Eric dies before Pat,
* Eric’s gross estate __ ____?? will / will not____ __ include the value of his remainder interest because it does not terminate at his death.
* Interest will be includable even if it is limited, contingent, or extremely remote as long as it does not end when the decedent dies.
* Of course, the contingency or remoteness of the interest will affect its valuation.

A

Brett gives Eric’s wife, Pat, the house in Miami for her lifetime and the remainder to Eric, and Eric dies before Pat,
* Eric’s gross estate will include the value of his remainder interest because it does not terminate at his death.
* Interest will be includable even if it is limited, contingent, or extremely remote as long as it does not end when the decedent dies.
* Of course, the contingency or remoteness of the interest will affect its valuation.

How well did you know this?
1
Not at all
2
3
4
5
Perfectly
11
Q

What is Income in respect of a decedent, or IRD?

A

Income in respect of a decedent, or IRD, is the right to future income earned but not received prior to a decedent’s death is a property interest that will be includable in the decedent’s estate.

Future income rights include:
* bonuses
* rents
* dividends
* royalties
* unpaid salary
* IRA accounts in excess of basis
* business accounts receivable
* vested amounts in qualified retirement plans
* interest payments
* the decedent’s share of any post-death partnership profits earned but not yet paid at death.

If a property is an IRD asset, not only will the value of the asset be included in the decedent’s estate, but the recipient of the income, whether the estate or a beneficiary, must pay the income tax liability.
* However, under IRC 691, to the extent any estate tax is paid on the income portion of an IRD asset, the person receiving this income may use this as an income tax deduction.

How well did you know this?
1
Not at all
2
3
4
5
Perfectly
12
Q

Example #1 (Taxation and Deductibility of IRD)

If in 2023, a decedent with a gross estate of $16.5 million bequeathed his IRA valued at $600,000 to his nephew,
* the value of the IRA __ ____?? would / would not____ __ be included in his gross estate.
* The decedent’s tentative tax would be $6,545,800 and the net federal estate tax would be $1,432,000 after using the decedent’s unified credit amount of $5,113,800 (2023) to offset the tentative tax.

However, suppose that the IRA had NOT been included in the decedent’s estate in 2023,
* then the uncle’s gross estate would only be worth $15.9 million, with a tentative tax of $6,305,800 and a net federal estate tax of $1,192,000.

The difference between the estate tax which included the IRA ($1,432,000) and the estate tax which did not include the IRA ($1,192,000) is $240,000.

  • If the nephew received a taxable distribution of $300,000 in 2023 from the IRA, he would report income of $__ ____?? would / would not____ __ and receive a deduction of $120,000. ($300,000 divided by $600,000 IRA x $240,000).
A

If in 2023, a decedent with a gross estate of $16.5 million bequeathed his IRA valued at $600,000 to his nephew,
* the value of the IRA would be included in his gross estate.
* The decedent’s tentative tax would be $6,545,800 and the net federal estate tax would be $1,432,000 after using the decedent’s unified credit amount of $5,113,800 (2023) to offset the tentative tax.

However, suppose that the IRA had NOT been included in the decedent’s estate in 2023,
* then the uncle’s gross estate would only be worth $15.9 million, with a tentative tax of $6,305,800 and a net federal estate tax of $1,192,000.

The difference between the estate tax which included the IRA ($1,432,000) and the estate tax which did not include the IRA ($1,192,000) is $240,000.

  • If the nephew received a taxable distribution of $300,000 in 2023 from the IRA, he would report income of $300,000 and receive a deduction of $120,000. ($300,000 divided by $600,000 IRA x $240,000).
How well did you know this?
1
Not at all
2
3
4
5
Perfectly
13
Q

IRD deductions are available on the estate income tax return Form _ ___??___ _

A

IRD deductions are available on the estate income tax return Form 1041 or may flow by way of Form K-1 from 1041.
* Deductions are allowed each year that IRD income is included in the beneficiary’s taxable income.
* The character of the IRD is taxed to the beneficiary as it would have been to the decedent.
* For example, ordinary income and capital gains are taxed at the beneficiary’s tax rate, and tax-exempt income is not taxable to the beneficiary.

How well did you know this?
1
Not at all
2
3
4
5
Perfectly
14
Q

Describe Gross Estate for Jointly Owned Property if held jointly w spouse

A

The decedent’s gross estate will include the value of his share of property held jointly with others.
* The amount included in the decedent’s gross estate will depend on whether the other joint owner is a spouse or not.

Property held jointly with spouses
* It is common for spouses to own property together as a tenancy-by-the-entirety or as a JTWROS.
* With a tenancy-by-the-entirety, property can only be owned between a husband and a wife. When the first spouse dies, 50% of the fair market value of the decedent spouse’s property is included in his gross estate. The surviving spouse inherits one-half of the decedent’s property and will own 100% of the property outright.
* The surviving spouse’s basis in the property will consist of the step-up in basis included in the decedent’s estate (50% FMV) in addition to the spouse’s original basis in the property, which does not receive a step-up when the first spouse dies.

How well did you know this?
1
Not at all
2
3
4
5
Perfectly
15
Q

Example (Stepped-Up Basis on Jointly Held Property)

A couple bought a home held as a tenancy-by-the-entirety for $60,000 each spouse has an acquisition basis of $30,000.
* If the home appreciates to $200,000 at the husband’s death, then $__ ____??____ __ will be included in the husband’s estate.
* The wife’s new basis in the home would be $__ ____??____ __.

A

A couple bought a home held as a tenancy-by-the-entirety for $60,000 each spouse has an acquisition basis of $30,000.
* If the home appreciates to $200,000 at the husband’s death, then $100,000 will be included in the husband’s estate.
* The wife’s new basis in the home would be $130,000.
* The same situation occurs when spouses own property as JTWROS.

How well did you know this?
1
Not at all
2
3
4
5
Perfectly
16
Q

Example (Stepped-Up Basis on Community Property)

For example, if a couple in Texas bought a home together for $20,000 and the FMV was $180,000 at the husband’s death, then
* $__ ____??____ __ was included in the husband’s estate
* the wife would inherit his one-half interest.
* Her original basis of $__ ____??____ __ would also be stepped-up to $__ ____??____ __ for a new basis in the home of $__ ____??____ __.

A

For example, if a couple in Texas bought a home together for $20,000 and the FMV was $180,000 at the husband’s death, then
* $90,000 was included in the husband’s estate
* the wife would inherit his one-half interest.
* Her original basis of $10,000 would also be stepped-up to $90,000 for a new basis in the home of $180,000.

Spouses who live in a community property state will include 50% of the FMV of the couple’s property in the decedent spouse’s gross estate, and the surviving spouse will inherit one-half of the property if it was left to the spouse in the will.
* The surviving spouse will have a complete step-up in basis at the first spouse’s death, because their one-half of the original basis and the decedent’s one-half of the property included in the gross estate are stepped up to FMV.

How well did you know this?
1
Not at all
2
3
4
5
Perfectly
17
Q

Describe Gross Estate for Jointly Owned Property with non-spouses

A

For property held as a JTWROS with a non-spouse, then the percentage-of-contribution rule would apply.
* The rule is that 100% of jointly-held property is includable in the estate of the first joint owner to die, except to the extent the survivor can prove contribution with funds that were not acquired by gift from the decedent.
* For example, if a brother paid 25% for a condo and his sister paid 75%, then when the brother dies only 25% of the FMV of the condo is included in the brother’s estate.

When property is owned with others as a tenancy-in-common, then only the deceased tenant’s fractional share of the property is included in the tenant’s estate.
* For example, four friends owned property together in different amounts. Roger owned 1/5 of the property therefore only 1/5 of the FMV of the property was included in Roger’s gross estate when he died.

How well did you know this?
1
Not at all
2
3
4
5
Perfectly
18
Q

Describe Gross Estate for Dower and Curtesy Interest (IRC 2034)

A

The value of the gross estate shall include the value of all property to the extent of any interest therein of the surviving spouse, existing at the time of the decedent’s death as dower or curtesy, or by virtue of a statute creating an estate in lieu of dower or curtesy.

Under the common law system, a surviving spouse has dower or curtesy rights.
* Dower is defined as the widow’s property rights under state law.
* Curtesy is defined as the widower’s property rights under state law.
* Community property states do not apply dower or curtesy rules.
* Most U.S. states have abolished the common-law dower and curtesy statutes and have in place other laws that treat both husband and wife identically, that is, the elective share, or statutory share statutes.
* However, what is important about IRC Section 2034 is the fact that the gross estate is not reduced by the value of any dower or curtesy interest.
* The value of these assets will be included in the decedent’s gross estate.

How well did you know this?
1
Not at all
2
3
4
5
Perfectly
19
Q

Describe Gross Estate for Certain Property Transferred (IRC 2035)

A

In general, a gift of property results in having that property permanently removed from the donor’s gross estate.
* Any future appreciation on that property is transferred to the new owners and will not be included in the donor’s gross estate either.

In some circumstances, gifts made within three years of death are included back into the donor’s gross estate to calculate the estate tax liability.
* However, this only applies for certain types of property transfers.
* In other words, not all property transferred within three years of death will have a value that is included in the decedent’s gross estate.
* It is important to know what types of property is permanently removed, and which types of property are included back in, under the “three year rule.

How well did you know this?
1
Not at all
2
3
4
5
Perfectly
20
Q

Which types of transfers would cause a gift made by the decedent to be included back in his gross estate?

A

There are certain situations that would cause a gift made by the decedent to be included back in his gross estate.

  • The donor created a life estate and gifted the remainder interest. The FMV of the life estate is included in the life tenant’s estate because the decedent had too much control over the property during lifetime and chose the remainder beneficiary of the property.
  • The donor kept a reversionary interest in property he gifted away. Because the donor retained the right to take the property back in the future, he retained too much control over the property and the value of the reversionary interest is included back in his estate.
  • This assumes that the reversionary interest is greater than 5% since the right to regain or dispose of the property must be worth actuarially more than 5% of the property’s value.
  • A grantor creates a revocable trust and transfers property to the trust. Because the grantor could revoke the trust, the value of the trust is included in his gross estate.

The donor or grantor can relinquish these property interests by giving up the life estate or the reversionary interest, or by making the revocable trust irrevocable.
* This will avoid inclusion in the grantor’s gross estate if he outlives that transfer for more than three years.

How well did you know this?
1
Not at all
2
3
4
5
Perfectly
21
Q

What types of transfers will the three-year date of death rule apply?

A

The types of transfers to which the three-year date of death rule will apply include the following:
* An interest in property that would be included in the gross estate under IRC Sections 2036, 2037, and 2038.
For example, the decedent relinquished a life estate in property or a right to lifetime income from a trust, within 3 years of death (Section 2036).
The decedent gave up a reversionary interest in property or trust corpus within 3 years of death (Section 2037).
The decedent relinquished a right to revoke, alter or amend a transfer within 3 years of death (Section 2038).

  • A transfer of a life insurance policy to which IRC Section 2042 would apply.
    For example, let’s assume A is the owner and the insured of a life insurance policy with a death benefit of $1 million.
    For estate planning purposes, it is recommended that the policy be gifted into an irrevocable life insurance trust, an ILIT. A dies a year after the policy has been gifted into the trust. What is included in A’s gross estate? Since A made a gift of the life insurance policy within 3 years of A’s death, the full $1 million death benefit of the policy would be included in A’s gross estate.
    Note that an owner of a life insurance policy who is not the insured can transfer this policy and not be subject to the 3-year rule.
  • Any gift tax liability paid within three years of the decedent’s death. For example, B makes a taxable gift of $1 million after using his $12,920,800 (2023) lifetime exemption, pays a gift tax of $400,000 ($1,00,000 x 0.40), and dies 2 years later. Although the $1 million gift is excluded from B’s gross estate, the $400,000 gift tax liability, which was paid, would be included in the gross estate.
How well did you know this?
1
Not at all
2
3
4
5
Perfectly
22
Q

Describe Gross Estate for Retained Life Estate (IRC 2036)

A

The gross estate includes the value of property the decedent has gifted away over which the decedent retained or reserved:
* The right to use, possess, or enjoy the property, or receive income during lifetime. (e.g., a mother creates a life estate in her home to live there until her death and gifts the remainder interest in her home to her son. The FMV of the property will be included in her gross estate at death.)
* The right to designate who would use, possess or enjoy the property. (e.g., the decedent, as trustee of an irrevocable trust, had discretion to decide which beneficiary could receive income or corpus from the trust.)

If either of the above rights had been retained by the decedent, in order for an asset to be included in the gross estate under 2036, the right must have been retained by the decedent for:
* Life
* Period not ascertainable without reference to death
* Period that does not end before the decedent’s death

How well did you know this?
1
Not at all
2
3
4
5
Perfectly
23
Q

Example (Full Rights Retained by Decedent)

Mom transfers 1,000 shares of stock to her daughter and reserves the right to receive the income from all 1,000 shares.
* When Mom dies, since she retained the right to the income from all 1,000 shares during her lifetime, __ ____??____ __% of the value of the shares will be included in Mom’s gross estate.

A

Mom transfers 1,000 shares of stock to her daughter and reserves the right to receive the income from all 1,000 shares.
* When Mom dies, since she retained the right to the income from all 1,000 shares during her lifetime, 100% of the value of the shares will be included in Mom’s gross estate.

How well did you know this?
1
Not at all
2
3
4
5
Perfectly
24
Q

Example (Partial Rights Retained by Decedent)

Mom transfers 1,000 shares of stock to her daughter and reserves the right to receive income from only 500 shares of stock.
* When Mom dies, since she only retained the right to receive income from 500 shares of stock, __ ____??____ __ will be included in her gross estate.

A

Mom transfers 1,000 shares of stock to her daughter and reserves the right to receive income from only 500 shares of stock.
* When Mom dies, since she only retained the right to receive income from 500 shares of stock, only the value of the 500 shares will be included in her gross estate.

How well did you know this?
1
Not at all
2
3
4
5
Perfectly
25
Q

What is the The Rationale for IRC 2036?

A

The rationale for including this type of lifetime transfer is that the right to enjoy or control property or designate who will receive the property or its income is characteristic of ownership.

How well did you know this?
1
Not at all
2
3
4
5
Perfectly
26
Q

Describe Gross Estate for Transfer at Death (IRC 2037)

A

The value of property, which has been transferred, must be included in the gross estate if the donee’s possession and enjoyment of the property is contingent upon surviving the decedent.

A Section 2037 transfer includes in the decedent’s gross estate the value of the property transferred if:
* The donee must survive the decedent.
* The decedent retained a reversionary interest in the property worth more than 5% of the value of the transferred property immediately before death.

How well did you know this?
1
Not at all
2
3
4
5
Perfectly
27
Q

Example #1 Application of Section 2037

Albert makes an irrevocable transfer into trust for his spouse. His spouse is to receive income from the trust for life. Upon the spouse’s death, the remainder of the trust will pass to Albert, if living, otherwise to Albert’s children. Albert dies in the current year.
What is included in Albert’s estate?
* The interest to Albert’s children __ ____?? would / would not____ __ be included in Albert’s estate because it is contingent upon the children surviving Albert.
* In other words, if his spouse dies, Albert has a reversionary interest in the property.
* Assuming the value of this reversionary interest satisfies the 5% requirement, the value of the reversionary interest __ ____?? would / would not____ __ be included in Albert’s estate.
* Since the spouse had a life estate in the income from the trust, the value of the spouse’s lifetime interest in the property __ ____?? would / would not____ __ be included in Albert’s gross estate.

A

Albert makes an irrevocable transfer into trust for his spouse. His spouse is to receive income from the trust for life. Upon the spouse’s death, the remainder of the trust will pass to Albert, if living, otherwise to Albert’s children. Albert dies in the current year.
What is included in Albert’s estate?
* The interest to Albert’s children would be included in Albert’s estate because it is contingent upon the children surviving Albert.
* In other words, if his spouse dies, Albert has a reversionary interest in the property. A
* ssuming the value of this reversionary interest satisfies the 5% requirement, the value of the reversionary interest would be included in Albert’s estate.
* Since the spouse had a life estate in the income from the trust, the value of the spouse’s lifetime interest in the property would not be included in Albert’s gross estate.

How well did you know this?
1
Not at all
2
3
4
5
Perfectly
28
Q

Example #2 Application of Section 2037

Brandi transfers property into a trust for her child’s benefit. The child is to receive income from the trust for a lifetime. Upon the child’s death, assets then remaining in the trust will transfer to the grandchildren, if alive, if not then to Brandi or Brandi’s estate. Brandi dies in the current year.
* What portion of this trust will be included in Brandi’s estate under Section 2037?

A
  • Since either the child or the grandchildren could take from the trust without surviving Brandi, the requirements of Section 2037 have not been satisfied.
  • Therefore, regardless of the reversionary interest to Brandi, there is no inclusion of assets in Brandi’s gross estate.

This is often called the “But if…back to…” section because it will not be operative unless the transferring document provides wording to the effect of: “But if the donee does not survive the donor, the property comes back to the donor.

How well did you know this?
1
Not at all
2
3
4
5
Perfectly
29
Q

Define Reversionary Interest

A

The right to regain the property is called a reversionary interest.
* To cause inclusion, the actuarial value of the transferor’s reversionary interest must be significant.
* Stated more precisely, the right to regain the property, or the right to dispose of it, must be worth actuarially more than 5% of the property’s value.

How well did you know this?
1
Not at all
2
3
4
5
Perfectly
30
Q

Example (Reversionary Interest Application)

During his lifetime, Stuart transferred property to his wife, Mona, for her lifetime. Upon Mona’s death, the property was to return to Stuart if he was living. If he was not living, the property was to go to Stuart’s daughter, Ellen. What is the outcome if Stuart dies before Mona?
* Stuart’s daughter can obtain possession or enjoyment of the property only if she survives Stuart.
* Stuart retained a reversionary interest under the original transfer. If Stuart’s reversion is worth more than 5% of the value of the property he placed in a trust, the value of the remainder interest will be includable in Stuart’s estate. Such a transfer is includable because it is considered to be, in substance, a substitute for disposing of the property by will.

A

During his lifetime, Stuart transferred property to his wife, Mona, for her lifetime. Upon Mona’s death, the property was to return to Stuart if he was living. If he was not living, the property was to go to Stuart’s daughter, Ellen. What is the outcome if Stuart dies before Mona?
* Stuart’s daughter can obtain possession or enjoyment of the property only if she survives Stuart.
* Stuart retained a reversionary interest under the original transfer. If Stuart’s reversion is worth more than 5% of the value of the property he placed in a trust, the value of the remainder interest will be includable in Stuart’s estate. Such a transfer is includable because it is considered to be, in substance, a substitute for disposing of the property by will.

How well did you know this?
1
Not at all
2
3
4
5
Perfectly
31
Q

Section 1 - Gross Estate Assets Summary

The gross estate, the starting point for the computation of the estate tax, is composed of the value of the decedent’s interest in all property. The types of property whose value is included in the gross estate include tangible and intangible, and real as well as personal. Additionally, outright ownership of property is not required for its value, or a portion thereof, to be included in the gross estate.

In this lesson, we have covered the following:
* Property owned outright: Defined in IRC Section 2033 as includable in the decedent’s gross estate if it was beneficially owned by the decedent and transferred at death by his or her will or state intestacy laws. It is only the value of the property owned by the decedent and transferred at the decedent’s death which will be included in the gross estate. This includes the value of all real and personal property. It does not include property in which the decedent possessed just the bare legal title but had no beneficial interest.
* Jointly owned property: If owned by spouses, then 50% of the FMV of the property is included in the deceased spouse’s estate. If owned by non-spouses, the percentage-of-the-contribution rule applies.
* Dower and Curtesy interest: The interest of the surviving spouse in the real property owned by the decedent under common law in certain states of the U.S. where it is applicable. IRC Section 2034 defines that this interest is includable in the decedent’s gross estate.

A
  • Three-year rule and exceptions: Certain property transferred as gifts within three years of the death of the decedent are not generally includable in gross estate according to IRC Section 2035. The exceptions to this rule are transfers in which the decedent had retained certain interest or power over the gifted property such as IRC Section 2036 retention of life estate, IRC 2037 transfers taking effect at death, IRC Section 2038 revocable gifts, and IRC Section 2042 life insurance policies.
  • Property with a retained life estate: Property identified under IRC Section 2036 is includable in the decedent’s gross estate. When a decedent gives away property during his or her lifetime but retains some control over or interest in the property, the property is generally included in the gross estate. The rationale behind the IRC is that the decedent retained the right to enjoy or control property or designate who will receive the property or its income. This right is characteristic of ownership.
  • Transfer at death: Property specified in IRC Section 2037 is includable in the decedent’s gross estate. The value of the property transferred and not the value of the interest retained will be includable. Property to which this section applies is that which the donee can begin to enjoy only by surviving the decedent. Since the decedent may have the right to regain the property during their lifetime, this right to regain is known as a reversionary interest.
How well did you know this?
1
Not at all
2
3
4
5
Perfectly
32
Q

The late Mr. Daniel Sherwood bequeathed his property to his niece as “To Sandra Sherwood for life, remainder to her issue and if no issue then to the American Red Cross.” Sandra died childless this year. At the time of Sandra’s death: (Select all that apply)
* Sandra’s life estate terminated.
* The property ownership was transferred to her issue according to her uncle’s will.
* Sandra did not own the property.
* The property interest must be included in her gross estate.

A

Sandra’s life estate terminated.
Sandra did not own the property.
* Under IRC Section 2033, no inclusion is required for property in which the decedent’s interest was obtained from someone else and was limited to lifetime enjoyment.
* Estate tax is a tax on the transfer of property at death. This means that if an interest terminated at the decedent’s death and the decedent had no right to transfer at death, its value will not be included.
* Sandra did not have anything that she could transfer.
* Her life estate, by its terms, terminated with her death so there was nothing to transfer.
* Though Sandra had an interest in the property at the time of her death, it was not a property interest that she could pass to someone else.

How well did you know this?
1
Not at all
2
3
4
5
Perfectly
33
Q

A When Sharon was 68, she was diagnosed as having terminal cancer. Doctors gave her a life expectancy of just eight months, with little hope of a cure. She had four children to whom she gifted the property that she owned outright. She died a year later. Which of her properties that she gifted as follows would be includable in the gross estate? (Select all that apply)
* Her villa to Stephanie, retaining life estate
* Her country house to Martha, for life, at Martha’s death, it returns to Sharon, unless Sharon is dead, then remainder to Martha’s issue
* Her shares in Oracle Corp., to Winston, retaining power to revoke
* Her garment factory to Leyland

A

Her villa to Stephanie, retaining life estate
Her country house to Martha, for life, at Martha’s death, it returns to Sharon, unless Sharon is dead, then remainder to Martha’s issue
Her shares in Oracle Corp., to Winston, retaining power to revoke

  • As per IRC Section 2035, gifts made within three years of death are not includable.
  • The exceptions are properties includable under IRC Sections 2036, 2037, 2038 or 2042.
  • The villa falls under Section 2036 because Sharon retains life estate.
  • The country house as life estate falls under Section 2037 because it is contingent that Martha be alive to enjoy the gift, with Sharon still retaining a right to regain the property personally.
  • Her shares in Oracle Corp., fall under Section 2038 because Sharon retained the power to revoke.
  • Therefore, these three properties-the villa, country house and Oracle shares-are includable in the gross estate.
  • The only property that is not includable is the garment factory, which was a complete gift.
How well did you know this?
1
Not at all
2
3
4
5
Perfectly
34
Q

A dower is the right of:
* The surviving wife to an interest for life in one-fourth of all real property owned by her husband during their marriage.
* The surviving wife to an interest for life in all of the real property owned by her husband during their marriage.
* The surviving husband to an interest for life in one-fourth of all real property owned by his wife during their marriage.
* The surviving husband to an interest for life in all of the real property owned by his wife during their marriage.

A

The surviving wife to an interest for life in all of the real property owned by her husband during their marriage.
* Under the common law system, marriage creates a spousal interest in the real property owned by the decedent.
* Dower is the right of the surviving wife to an interest, for life, in all of her husband’s real property.
* The right of the husband to an interest for life in all of the real property owned by his wife during their marriage is called curtesy.

How well did you know this?
1
Not at all
2
3
4
5
Perfectly
35
Q

Section 2 - Determining the Value of Gross Estate

Let us continue with the remaining Sections of the IRC dealing with the assets to be included in the gross estate. Remember, unless the IRC specifically excludes property, the total value of all assets or interests in assets over which the decedent held an element of control will be included in the decedent’s gross estate. Additionally, these assets include the treatment of property that has been transferred over that the decedent retained the right to revoke and assets over which the decedent held a general power of appointment. We will also cover how annuities, life insurance, and jointly-owned assets are treated for gross estate inclusion purposes.

To ensure that you have a thorough understanding of other IRC Sections, the following topics will be covered in this lesson:
* Revocable Transfer (IRC 2038)
* Annuities (IRC 2039)
* Jointly-Owned Property (IRC 2040)
* Power of Appointment (IRC 2041)
* Life Insurance (IRC 2042)
* Estate of Surviving Spouse where Marital Deduction previously allowed (IRC 2044)

A

Upon completion of this lesson, you should be able to:
* Identify a revocable transfer
* Define an annuity
* Identify types of annuities
* List the qualifications that govern the inclusion of annuities in the gross estate.
* Describe the 50-50 rule applied to jointly-owned property
* Explain the percentage-of-contribution rule and its use
* Define power of appointment and its treatment in valuing the gross estate
* Specify the conditions under which life insurance is includable in the gross estate
* Discuss how a previously utilized marital deduction impacts the gross estate of the surviving spouse

How well did you know this?
1
Not at all
2
3
4
5
Perfectly
36
Q

Describe Gross Estate for Revocable Transfer (IRC 2038)

A

A revocable transfer is a transfer of property in which the decedent retained the right to alter, amend, revoke or terminate the gift.
* When a grantor creates an irrevocable trust but can decide whether to accumulate trust income or distribute it to trust beneficiaries, the grantor retains a right to alter or amend a transfer.
* When a grantor establishes a revocable trust, the grantor retains a right to revoke or terminate a transfer.

The gross estate of the decedent will include the value of the property subject to this power.

IRC Section 2038 would apply regardless of whether the decedent retained the said power alone or together with others. Only the value of property subject to this power will be included in the gross estate, not the entire value of the transferred property.

How well did you know this?
1
Not at all
2
3
4
5
Perfectly
37
Q

What’s Included in Gross Estate for Revocable Transfer (IRC 2038)

A

The courts have broadly interpreted revocable transfers. For example, the mere power retained by the donor to vary the timing of when a beneficiary will receive an interest will cause inclusion, even if the beneficiary cannot forfeit the interest.

There are, however, exceptions for inclusion in the gross estate for transfers that would otherwise qualify as revocable transfers. These exceptions include the following:
* If the decedent’s power can only be exercised with the consent of all of the parties having an interest in the transferred property, and
* If the power retained by the decedent adds nothing to the rights of the parties under local law, or
* If the prohibited power is retained, it is subject to an ascertainable standard.

An ascertainable standard is an external standard that limits the decedent’s exercise of a power for the health, education, maintenance, or support (HEMS) of a beneficiary.
* Therefore, even if the decedent held a prohibited power, if this power was limited by an ascertainable standard, the value of the assets over which this power was held would not be included in the decedent’s gross estate.

How well did you know this?
1
Not at all
2
3
4
5
Perfectly
38
Q

Example #1 (Estate Inclusion Scenario)

Larry creates a trust with Larry as the trustee. Larry has retained the power to revoke the trust. Larry has transferred $1 million of property into the trust. Larry dies. Since Larry has the right to revoke the trust,
* __ ____??____ __ of property will be included in Larry’s estate.

A
  • the full $1 million of property will be included in Larry’s estate.
How well did you know this?
1
Not at all
2
3
4
5
Perfectly
39
Q

Example #2 (Estate Inclusion Scenario)

Larry creates a trust with Erica as the trustee. Larry retains the power to name a successor trustee or to remove a current trustee and appoint a new trustee including himself. Larry has transferred $1 million into the trust. Larry dies.
* Since Larry had the right to remove and appoint a new trustee, including Larry, how much will be included in Larry’s estate?

A
  • Since Larry had the right to remove and appoint a new trustee, including Larry, which may have impacted the beneficiaries’ right to enjoy the timing of the trust benefits, the full $1 million will be included in Larry’s estate.
How well did you know this?
1
Not at all
2
3
4
5
Perfectly
40
Q

Example (Ascertainable Standard Application)

Alice transfers property into trust with income to Bradley, remainder to Coretta. Alice retains the power to invade the trust principal for Bradley’s health, support, maintenance, and education (the ascertainable standard).
* Even though Alice has retained the right to affect the timing of Bradley’s enjoyment of the property, the value of the property over which Alice retains the power to revoke __ ____?? will / will not____ __ be included in Alice’s gross estate.

A
  • Even though Alice has retained the right to affect the timing of Bradley’s enjoyment of the property, since it is limited by an ascertainable standard, the value of the property over which Alice retains the power to revoke will not be included in Alice’s gross estate.

There are, however, exceptions for inclusion in the gross estate for transfers that would otherwise qualify as revocable transfers. These exceptions include the following:
* If the decedent’s power can only be exercised with the consent of all of the parties having an interest in the transferred property, and
* If the power retained by the decedent adds nothing to the rights of the parties under local law, or
* If the prohibited power is retained, it is subject to an ascertainable standard.

An ascertainable standard is an external standard that limits the decedent’s exercise of a power for the health, education, maintenance, or support (HEMS) of a beneficiary.
* Therefore, even if the decedent held a prohibited power, if this power was limited by an ascertainable standard, the value of the assets over which this power was held would not be included in the decedent’s gross estate.

How well did you know this?
1
Not at all
2
3
4
5
Perfectly
41
Q

Describe Gross Estate Calculation for Annuities (IRC 2039)

A

The general rule is that the gross estate includes the present value of an annuity or other payment receivable by a beneficiary as a result of surviving the decedent.

An annuity is defined as payments, or the decedent’s right to receive payments, that do not terminate upon the decedent’s death.
* In other words, they include periodic payments over a specified period of time under an enforceable contract where the decedent had the right to the payment.
* An annuity under IRC Section 2039 also includes the decedent’s right to receive future benefits.

How well did you know this?
1
Not at all
2
3
4
5
Perfectly
42
Q

What is an Annuity?

A

An annuity is a contract entered into whereby one party agrees to pay another, the annuitant, a series of payments over a period of time.
* An annuity is defined as a systematic liquidation of principal and interest.
* It might be a life annuity where payments are made for the life of the annuitant, or it might be a term certain annuity where payments are made over a period of years.
* A very common example of an annuity is pension income.

How well did you know this?
1
Not at all
2
3
4
5
Perfectly
43
Q

Go to http://www.sec.gov/answers/annuity.htm and read the page.

After reviewing the above link, you should be able to answer the following questions:

What are the savings advantages of an annuity?
What are the advantages and disadvantages of an annuity?

A

An annuity is a contract between you and an insurance company that is designed to meet retirement and other long-range goals, under which you make a lump-sum payment or series of payments. In return, the insurer agrees to make periodic payments to you beginning immediately or at some future date.

Annuities typically offer tax-deferred growth of earnings and may include a death benefit that will pay your beneficiary a specified minimum amount, such as your total purchase payments.
* While tax is deferred on earnings growth, when** withdrawals are taken from the annuity, gains are taxed at ordinary income rates, and not capital gains rates**.
* If you withdraw your money early from an annuity, you may pay substantial surrender charges to the insurance company, as well as tax penalties.

There are generally three types of annuities — fixed, indexed, and variable. In a fixed annuity, the insurance company agrees to pay you no less than a specified rate of interest during the time that your account is growing. The insurance company also agrees that the periodic payments will be a specified amount per dollar in your account. These periodic payments may last for a definite period, such as 20 years, or an indefinite period, such as your lifetime or the lifetime of you and your spouse.

In an indexed annuity, the insurance company credits you with a return that is based on changes in an index, such as the S&P 500 Composite Stock Price Index.

In a variable annuity, you can choose to invest your purchase payments from among a range of different investment options, typically mutual funds. The rate of return on your purchase payments, and the amount of the periodic payments you eventually receive, will vary depending on the performance of the investment options you have selected.

Variable annuities are securities regulated by the SEC. An indexed annuity may or may not be a security; however, most indexed annuities are not registered with the SEC. Fixed annuities are not securities and are not regulated by the SEC. You can learn more about variable annuities by reading our Updated Investor Bulletin: Variable Annuities.

How well did you know this?
1
Not at all
2
3
4
5
Perfectly
44
Q

Describe Gross Estate Calculation for Annuities (IRC 2039)

A

Annuities are broadly classified into two types:
* Commercial annuities
* Private annuities

The person or entity who assumes the obligation for a private annuity is not in the business of selling annuities, in contrast to commercial annuities.
* Further, commercial annuities are usually funded with cash, while private annuities are often funded with various types of property, such as real estate or corporate stock.
* Commercial annuity companies use standard actuarial tables to determine their payout terms, whereas a private annuity may determine its payout terms according to many other factors.

If the decedent owned a commercial annuity on the date of death, the gross estate would include the cost of a comparable contract sold by the issuing insurance company on the decedent’s date of death.
* (In other words, the replacement value of the annuity.)
* The value of the survivorship interest in a commercial annuity is the amount that the same insurance company would charge for a single annuity on the survivor at the time of the first annuitant’s death.
* The value of survivorship in a private annuity is determined using government valuation tables.

How well did you know this?
1
Not at all
2
3
4
5
Perfectly
45
Q

Example (Present Value of an Annuity)

Steve purchased a private annuity that would pay him $20,000 a year for life and, upon his death, would pay his wife, Jayne, $10,000 a year for as long as she lives.
* If Steve died before Jayne this year, the present value of future annuity payments to Jayne __ ____?? would / would not____ __ be includable in his gross estate.
* Using the government valuation table called Table S, if Jayne were aged 55 when Steve died, and a 6% rate (as specified in Section 7520) was applicable, the annuity would have a present value at that time of $118,459.

A

Steve purchased a private annuity that would pay him $20,000 a year for life and, upon his death, would pay his wife, Jayne, $10,000 a year for as long as she lives.
* If Steve died before Jayne this year, the present value of future annuity payments to Jayne would be includable in his gross estate.
* Using the government valuation table called Table S, if Jayne were aged 55 when Steve died, and a 6% rate (as specified in Section 7520) was applicable, the annuity would have a present value at that time of $118,459.

This inclusion provision applies not only to commercial, joint and survivor annuities but also to certain other types of payments made under a contract or agreement to the survivor(s) of the decedent.

How well did you know this?
1
Not at all
2
3
4
5
Perfectly
46
Q

What three qualifications is the general rule regarding annuities (IRC Section 2039) subject to?

A

The general rule regarding annuities, as defined in IRC Section 2039, is subject to three qualifications:
* Contracts such as a life annuity that provide payments to the decedent and end at death are not subject to this provision because they are not capable of death time transmission, that is, there is no transferable interest.
* If the survivor or anyone other than the decedent furnished part of the original purchase price, then that portion of the survivor’s annuity will not be included in the decedent’s gross estate. So if the survivor paid one-third of the initial premium, only two-thirds of the value of survivor’s income interest would be includable. If the decedent’s employer furnished all or part of the purchase price, that contribution is treated as if it were made by the decedent.
* Where the death proceeds of a life insurance policy are taken under a settlement option, they are considered life insurance proceeds rather than an annuity and are not taxed according to annuity rules.

47
Q

What are the two rules that affect the estate taxation of property held jointly with the right of survivorship?

A

Jointly-Owned Property (IRC 2040)

There are two rules that affect the estate taxation of property held jointly with the right of survivorship:
* When the joint tenants are spouses
* When the joint tenants are not spouses

48
Q

Describe Gross Estate Calculation for Assets owned jointly with Spouse (IRC 2039)

A

If an asset is owned jointly with rights of survivorship by a married couple, or as a tenancy by the entirety, upon the death of the first spouse only 50% of the value of the jointly-owned property will be included in the first-to-die spouse’s gross estate.
* This is regardless of the size of their contribution. In fact, even if a contribution of more than 50% can be proven by the survivor, this rule must be used.

This rule applies to both real and personal property regardless of how it was acquired or when it was purchased.

49
Q

Describe Gross Estate Calculation for Assets owned jointly with non-spouse (IRC 2039)

A

The percentage-of-contribution rule, also known as the consideration-furnished rule, affects the estate taxation of property held jointly with the right of survivorship and is used where the joint owners are not spouses, or if one of the spouses is not a U.S. citizen.
* Essentially, this provision measures the estate tax includability of jointly-held property with survivorship rights by referring to the portion of the purchase price attributable to the decedent’s contribution.

Actually, the rule is that 100% of jointly-held property is includable in the estate of the first owner to die unless the survivor can prove contribution out of funds other than those acquired by gift from the decedent.
* So, if the survivor can prove a contribution of one-third of the original purchase price, only two-thirds of the value of the jointly-held property will be includable in the decedent’s estate.
* If the survivor can prove a contribution of two-thirds of the original purchase price, only one-third would be includable in the decedent’s estate.

50
Q

Describe Gross Estate Calculation for Power of Appointment (IRC 2041)

A

The gross estate includes the value of property interests over which the decedent had a general power of appointment.
A power of appointment determines who will own or enjoy the property subject to the power and when they will own or enjoy it.

Power of appointment is the right to say who is to receive property in trust. When a power of appointment is so extensive that it approaches actual ownership of the affected property, this is known as a general power of appointment.
* In other words, a general power of appointment allows the decedent to transfer the property to himself, his estate, his creditors or the creditors of his estate.
* Since this is similar to outright ownership over the property, property subject to a general power will be includable in the gross estate of the person who holds the power.
* Such property will be includable in a decedent’s gross estate regardless of whether the power could be exercised at death or only during lifetime or whether the power of appointment was ever exercised by the holder of the power.

A general power of appointment is a power to appoint property to the powerholder, the powerholder’s estate, or the creditors of either.
* If a decedent held a general power of appointment over property on his or her death, the value of the property is included in the powerholder’s gross estate.

51
Q

Example (General Power of Appointment)

Daniel may be receiving a lifetime income under a trust and have the power to withdraw all or a portion of the trust corpus during his lifetime. This right to invade the trust corpus is a general power of appointment and will cause the entire value of the trust to be included in Daniel’s gross estate on death, even though he may never have actually exercised the withdrawal right.

A

Daniel may be receiving a lifetime income under a trust and have the power to withdraw all or a portion of the trust corpus during his lifetime. This right to invade the trust corpus is a general power of appointment and will cause the entire value of the trust to be included in Daniel’s gross estate on death, even though he may never have actually exercised the withdrawal right.

52
Q

Describe Gross Estate Calculation for Limited/Special Power of Appointment

A

While a general power of appointment enables a person to exercise it in favor of self, creditors, his or her estate or creditors of the estate, a special or limited power of appointment enables the individual to appoint anyone other than self, his or her estate, or creditors to receive property.

However, there are three situations that allow the holder to appoint property to himself or herself, and still have the power of appointment considered a special power of appointment.
* The holder may use trust property for himself but only with the donor’s consent.
* The holder may use trust property for himself only with the consent of the remaining trust beneficiaries, who have an adverse interest in the trust. The remaindermen have an adverse or unfavorable interest in the trust because the holder can exercise the power for his own benefit, which reduces the amount available to the remaindermen.
* The holder can exercise the power according to an ascertainable standard.

An ascertainable standard means that trust income and principal can be used for the holder’s health, education, maintenance, and support (HEMS).
* If the trust document says that the holder can use trust property for health, education, and “comfort,” or any wording other than HEMS, the power of appointment will be considered a general power instead of a special power.

Property over which an individual has such special or limited power of appointment will not be included in the holder’s gross estate.

53
Q

Are there instances where, even if the decedent holds a general power of appointment (also known as a GPA) over property, upon death, there will be no inclusion in the gross estate?

A

There are certain instances where, even if the decedent holds a general power of appointment (also known as a GPA) over property, upon death, there will be no inclusion in the gross estate.

Example (GPA: No Inclusion in the Gross Estate)

If the decedent’s GPA were limited to a power to appoint which does not exceed the greater of $5,000 or 5% of the value of the trust assets on an annual, noncumulative basis, these assets would not be included in the decedent’s gross estate.

Inclusion in the gross estate of property over which a decedent held a general power of appointment will be required in certain circumstances even if the power is released, meaning formally given up, or in some cases, if the power is allowed to lapse, that is, fail through the lack of use.

54
Q

Example (GPA: Inclusion in the Gross Estate)

For example, if the decedent who had a GPA limited to the noncumulative right to withdraw the greater of $5,000 or 5% of the value of the trust (known as the 5x5), dies and failed to exercise his right of withdrawal, the decedent’s gross estate would only include the value of the property the decedent could appoint to himself. In this instance, only the greater of $5,000 or 5% of the value of the trust assets would be included in his estate, reduced by any amount the holder exercised (already used) in the year he died.
* However, if the power to appoint exceeds the greater of 5x5, then the holder of the power is making a gift or transfer of the excess to the trust when he lets the power lapse.
* The amount included in the holder’s estate is the value of the assets subject to the power at the holder’s date of death.
* In addition, the holder’s estate will include a percentage equal to the accumulated amounts that the holder could have exercised each year, that were in excess of 5x5.

A

Inclusion in the gross estate of property over which a decedent held a general power of appointment will be required in certain circumstances even if the power is released, meaning formally given up, or in some cases, if the power is allowed to lapse, that is, fail through the lack of use.

55
Q

Describe Gross Estate Calculation for Incidents of Ownership (IRC 2042)

A

The gross estate includes the value of the death benefit proceeds of any life insurance policies in which the decedent held any incidents of ownership.

The term incidents of ownership refers to the right to benefit from the policy or decide who is to enjoy the benefit in an economic sense. For example, if an owner/insured had the right to name a beneficiary, surrender a policy, or borrow its cash value, the owner/insured would be enjoying the economic benefits of a policy.

Almost any meaningful ownership attribute in the policy held by the decedent will cause the entire death benefit proceeds to be includable in the owner/insured’s gross estate. This is true regardless of how the decedent obtained the ownership rights or whether the use of such powers could directly or indirectly benefit the decedent. Mere possession of the ownership right at death is all that is required to cause inclusion. The capacity to utilize the power is not required.

A decedent’s gross estate will also include the proceeds of any policy payable to his or her executor or for the benefit of the decedent’s estate, whether or not the decedent had retained any incidents of ownership in the policy at death.
* Therefore, naming the insured’s estate or executor, as either a direct or indirect beneficiary of the life insurance, will cause proceeds to be included in the gross estate.

In many cases, life insurance policies are owned by irrevocable trusts, also known as ILITs. The objectives of these trusts include: removing the death benefit proceeds from inclusion in the insured’s gross estate and providing the cash needed to assist the executor for the estate to pay the estate tax liability.
* However, if the ILIT contains a provision which requires the trustee to use the life insurance proceeds to pay for the expenses associated with the insured’s death, the proceeds of this life insurance, even if owned within the trust, will be included in the insured’s estate.
* Therefore, with these types of trusts, it is imperative that the trustee be given the discretion to deal with the personal representative of the estate.

56
Q

Describe Gross Estate Calculation for Corporate-Owned Life Insurance (COLI)

A

Corporate-owned life insurance (COLI) is includable in a decedent’s estate if he or she owned more than 50% of the corporation’s stock at death and is the insured on the policy, but only to the extent that it is payable to a party other than the corporation or its creditors.

57
Q

Example (Tax Applications of COLI)

Assume Chuck Heikenen owned 60% of the CH Corporation. The corporation owned a $1 million policy on Chuck’s life.
* If the $1 million were payable to the corporation, it would __ ____??____ __.
* However, if the $1 million were payable to Chuck’s brother, the __ ____??____ __ would be includable.
* The Internal Revenue Service (IRS) may also claim that the distribution constituted either a __ ____??____ __ and apply income tax to the distribution as well.

A

Assume Chuck Heikenen owned 60% of the CH Corporation. The corporation owned a $1 million policy on Chuck’s life.
* If the $1 million were payable to the corporation, it would push up the value per share of the stock included in Chuck’s estate.
* However, if the $1 million were payable to Chuck’s brother, the entire $1 million would be includable.
* The Internal Revenue Service (IRS) may also claim that the distribution constituted either a dividend or deferred compensation and apply income tax to the distribution as well.

58
Q

Describe Gross Estate Calculation for Surviving Spouse’s Estate (IRC 2044)

A

In the case of a married couple, if the first spouse were allowed a marital deduction for a qualifying income interest left to the surviving spouse, upon the death of the surviving spouse the value of the property in which the surviving spouse had a qualifying income interest would have to be included in his or her estate.

The result is the same regardless of when the qualifying income interest was created: during the first spouse’s lifetime, or after the first spouse’s death.

The key to IRC Section 2044 is to ensure that property that transfers to a surviving spouse utilizing the marital deduction will be included in the gross estate of the surviving spouse.
This section makes sense if we consider that the marital deduction simply postpones the payment of the estate tax until the death of the surviving spouse.

59
Q

Example (Surviving Spouse’s Estate)

If a $1,000,000 marital deduction was allowed at Sam’s death for property left to his wife, Sadie, at Sadie’s death the value of such property at the time of her death will be included in Sadie’s estate.
* However, if the election were made to transfer the same $1 million into a QTIP (qualified terminable interest property) trust, the executor must agree in the election to include the QTIP assets then remaining in Sadie’s estate.

A
60
Q

Section 2 - Determining the Value of Gross Assets Summary

There are some Sections in the IRC that govern specific assets such as annuities, life insurance, properties jointly-owned, and properties in which the decedent retained certain rights. It is important that financial planners can identify what property is included in a client’s gross estate and can estimate the value of that property at the client’s death. These calculations can inform the financial planner whether the client has enough resources and liquidity to pay for estate settlement costs and distribute property to loved ones as planned.

In this lesson, we have covered the following:
* Revocable transfer of property: Refers to a transfer made during the lifetime of the decedent in which he or she retained a power to alter, amend, revoke or terminate the gift. According to IRC Section 2038, the value of property subject to that power will be included in the decedent’s gross estate. This is applicable even if the owner holds these powers as a trustee and cannot regain the property or personally benefit from it in any way.
* Annuities: An annuity is a systematic liquidation of principal and interest. Under IRC Section 2039, the value of an annuity is included in gross estate if the payments do not end at death but are receivable by a beneficiary by reason of surviving the decedent. The includable portion of the payment is the portion of the value attributable to the amounts paid by the decedent or the decedent’s employer on the decedent’s behalf.

A
  • Jointly-owned property: Includable in the gross estate depending on who are the joint owners. In the case of spouses, upon the death of the first spouse, one-half of the value of the jointly-owned asset will be included in that spouse’s gross estate. In the case of nonspouse joint owners, 100% of the value of the jointly-owned property will be included in the first owner’s estate unless the surviving owner can show contribution toward the purchase of the asset.
  • Power of appointment: A right to determine who will own or enjoy the property and when they will do so. A general power of appointment enables a person to exercise it in favor of self, creditors, his or her estate or creditors of the estate. In contrast, a special or limited power of appointment enables the individual to appoint anyone other than self, his or her estate or creditors to receive property. Under IRC Section 2041, property subject to a general power of appointment is included in the decedent’s gross estate, while property over which an individual has special or limited power of appointment is not includable.
  • Life insurance: Proceeds on a decedent’s life payable to the decedent’s estate or to any other beneficiaries are included in the gross estate if the decedent retained incidents of ownership. This is defined in IRC Section 2042. Incidents of ownership include the right to benefit from the policy or the power to change the beneficiary.
  • Surviving spouse’s gross estate: Includes the value of any qualifying income interest property for which the first spouse’s estate had taken a marital deduction.
61
Q

David transferred his estate to his daughter, Jennifer, in 2015 with the condition that she would receive income until age 35, at which age she would receive the principal if she were not married. He retained the power to distribute the income to his son, Stephan if Jennifer should get married. David dies in 2023 when Jennifer is 32 years old and not married.
Which of the following is includable in David’s gross estate?
* The value of the remaining income term interest for three years from David’s date of death to when Jennifer reaches age 35.
* The value of the principal.
* The value of both the principal and all interest paid to Jennifer from 2015 to David’s date of death.
* The value of both the principal and the remaining income term interest for three years from David’s date of death to when Jennifer reaches age 35.

A

The value of the remaining income term interest for three years from David’s date of death to when Jennifer reaches age 35.
* According to IRC Section 2038, in David’s case only the transfer of income is a revocable transfer.
* Only the value of property subject to this power will be included in the gross estate and not the entire value of the transfer of property.
* The property for which David retained this power at the time of his death was the remaining income that would be paid to Jennifer until she reaches age 35.
* David did not retain the right to the transfer of principal because Jennifer would receive it in any case at age 35, and therefore the principal is not includable.

62
Q

Tom purchased an annuity contract for his son, with payments to be commenced at his death, which was irrevocable. When Tom dies, which of the following apply to the value of the annuity interest? (Select all that apply)
* Includable in Tom’s gross estate because he purchased the annuity.
* Includable in Tom’s gross estate because a transfer of property takes place at death.
* Not includable in Tom’s gross estate because he has no beneficial interest in the payments.
* Not includable in Tom’s gross estate because he has no right to payments.

A

Not includable in Tom’s gross estate because he has no beneficial interest in the payments.
Not includable in Tom’s gross estate because he has no right to payments.
* Both IRC Sections 2039 and 2033 are applicable here.
* Tom does not have any beneficial interest in the payments because they will commence at his death and go to his son.
* The annuity is not being transferred at death. Therefore, it is not includable in Tom’s gross estate.
* According to IRC Section 2039, it will be includable only if it provided the decedent with a payment or a right to payment for life.

63
Q

Benjamin and his son Jerome jointly purchase 2,500 shares of Petra Inc. at $4/share. Benjamin paid $7,500, while Jerome paid the remaining $2,500 required for this purchase. At the time of Benjamin’s death this year, the stock is worth $20 per share. How much will be includable in his gross estate?
* $37,500
* $7,500
* $10,000
* $5,000

A

$37,500

  • The percentage-of-contribution rule will be applied here because the property is held jointly with the right of survivorship, and the parties are non-spouses.
  • This provision measures the estate tax by referring to the portion of the purchase price attributable to Benjamin’s contribution, which was 75%.
  • Therefore, the value includable in his gross estate is 75% of the market value of the shares when he dies, that is, 75% of $20 x 2,500, which is $37,500.
64
Q

Section 3 - Valuation Planning

The process of calculating estate taxes begins with determining the value of the gross estate. For a financial planner, valuation of assets during the estate planning process is important in order to:
* Determine the liquidity needs of the estate,
* Arrange for the funding of buy-sell agreements, and
* Determine what would be due as tax.

Generally, the gross estate includes the date of death fair market value of any asset included in the decedent’s gross estate. Therefore, the calculation of property interest, depending on the property type, consists of several variables. For example, a qualified appraiser’s skill and experience may significantly impact the valuation of certain estate assets.

The exception to the general date of death valuation rule is the alternate valuation rule. Under the alternate valuation rule, the decedent’s gross estate is the fair market value of the decedent’s assets six months after the date of death. Additionally, the law has special valuation provisions for certain types of property.

A

To ensure that you have a thorough understanding of valuation planning, the following topics will be covered in this lesson:
* Defining Valuation Planning
* Valuation of Real Property
* Valuation of Life Insurance
* Valuation of Annuities

Upon completion of this lesson, you should be able to:
* Define valuation of estate assets
* List the requirements and tax implications of valuation
* List the general valuation rules
* Describe how value is determined and its related problems
* Explain the significance of the valuation date
* Identify the factors that affect the valuation of real property
* Define special valuation and its qualification requirements
* Specify the method for valuation of life insurance
* Explain the valuation of various types of annuities

65
Q

Describe Transfer Tax Implications

A

Valuing property for federal gift, estate, and generation-skipping transfer tax purposes is a complex process.
* Frequently, taxpayers arrive at valuations that differ widely from the values established by the IRS.
* Courts will then be asked to resolve the valuation question.

Thus, value is a variable upon which reasonable minds can, and will, continue to differ. The use of careful and thorough appraisals by qualified experts, documentation of sales of similar property recently sold, and well-drawn arm’s length restrictive agreements, such as buy-sell arrangements, have proved effective tools in substantiating favorable values.
* An appraisal is not needed for all estate assets.
* You do not need an appraiser for publicly traded securities unless the decedent owned a significant portion of the outstanding shares.
* You do not need appraisals for cash or even for lump sum values of retirement plan assets.

66
Q

Describe the General Valuation Rules

A

According to the general rules, assets included in the gross estate are valued on the decedent’s date of death.

IRC Section 2031 deals with the factors which must be recognized in order to value items includable in the gross estate for federal estate tax purposes. Not only must all the factors affecting value be considered, but there must be sound reasoning for the relative weight given to each one.

67
Q

Describe IRC Section 2031

A

Under the IRC Regulations, value is the fair market value (FMV), “the price at which the property would change hands between a willing buyer and a willing seller, neither being under any compulsion to buy or to sell and both having reasonable knowledge of relevant facts.”

Thus, the value a particular person would place on property may vary greatly from the measure of worth placed on that same item by the government. In fact, it is neither necessary that there be an established market for an item, nor that there be the “willing buyer and seller” spoken of in the IRC Regulations. In the absence of an actual sale, the value is based on a hypothetical sale.

Generally, when an organized market does in fact exist, the market price will prevail. The ignorance of a material fact by the buying public or its inability to properly assess the significance of certain events does not form a basis for reducing the price indicated on the organized market.
* The contrary result applies in the absence of an established market.
* A purchase or sale is not regarded as determinative of value in situations in which one of the parties was ignorant of a material fact.

68
Q

Describe IRS Value Determination

A

The IRS would most likely consider all the facts and circumstances that a hypothetical buyer and seller would consider.
The derived price at which the property would have changed hands between two parties thus determines the IRS’s valuation.
By this rule, a forced sale or a sale outside of regular business channels would not be determinative of the value.
Thus, generally, the IRS’s position is that the price at which the item or a comparable item would have been sold at retail is determinative.

69
Q

Describe Valuation Problems

A

Value is basically a question of fact in those situations where there is an established market for identical property.
However, valuation problems become essentially problems of evidentiary proof, and often of opinion, where:
* There are different markets for the same property.
Example: A property with both wholesale and retail markets.
* The appraisal of the worth must be made on the basis of comparison with somewhat similar property.
For instance, the properties that must be selected and the comparability of the properties are some matters that must be considered. What is derived is at best an opinion based upon fact.
* The property in question is unique, such as a patent or copyright.
Here, the data must be analyzed and an opinion must be formulated as to how much the potential anticipated benefits are worth. It requires the examiner to be capable of making an adequate analysis.

In practice, the IRS and the courts frequently view valuation problems as problems of negotiation and compromise. Hence, the appraiser’s objective must be to derive a fair and sound value. If litigated, this value should be sustained by the court. Evidence and proof secured in the form of expert advice through appraisal, promptly after death, would likely have a greater probative value than evidence obtained at a later date.
As to the valuation of specific types of property, one must turn to the regulations, rulings, and court decisions.

70
Q

Describe the Valued Date

A

The alternate valuation date (AVD) is six months after death.
* The executor can elect the alternate valuation date only if the value of the decedent’s gross estate has diminished in value in six months and the estate tax liability is also less.
* If all of the decedent’s property is transferred to the surviving spouse by will, then the estate tax liability is zero and cannot decline, so the alternate valuation date could not be used.
* Once the date of death or alternate valuation is selected for valuation purposes, such date applies to all assets in the estate.

If the alternate valuation date is selected and if the property is distributed, sold, exchanged, or otherwise disposed of within six months of the decedent’s death, it will be valued as of that date, not the six month date.
* Certain types of property diminish in value as time goes on. For instance, the present value of an annuity reduces each time a payment is made.
* Any such property interest or estate whose value is affected by the mere passing of time is valued as of the date the decedent died.

71
Q

Johann passed away on 3/15/2023 with the following assets in his estate:
3/15/2023 6/15/2023 9/15/2023
Primary Residence $750,000 $715,000 $760,000
Investment Portfolio $250,000 $175,000 $205,000
His primary residence was sold on 6/15/2023. The executor made an alternate valuation election. Calculate the value of Johann’s gross estate.
* $1,000,000
* $965,000
* $890,000
* $920,000

A

$920,000
* By making an alternate valuation date election, the estate will be valued based on the FMV of the assets 6 months from the date of death.
* If an asset is sold or distributed within the 6-month window, the sale price is used for valuation.
* Therefore, the sale price of the residence on 6/15/2023 [$715,000] is added to the value of the investment portfolio on 9/15/2023 of [$205,000] for a total of $920,000.

72
Q

The greater of what 2 things will control the valuation In the absence of a market for such property?

A

Due to the uniqueness of land, the value of any real property as of a given date may be subject to widely differing opinions.

In the absence of a market for such property, the greater of the following will control the valuation:
* The highest price available, or
* The salvage value.

Where there is a market for real property, however, there are several factors that affect valuation.

73
Q

What are the Factors That Effect Valuation?

A

The basic factors that affect the valuation of real property, where there is a market for the property, are listed below:
* The nature and condition of the property, its physical qualities and defects, and the adequacy or inadequacy of its improvements.
* The size, shape, and location of the property.
* The actual and potential use of the property and how the trends of development and economic conditions, such as population growth, affect it.
* How suitable the property is for its actual or intended use.
* Zoning restrictions.
* Size, age, and condition of the buildings, including the degree of deterioration and obsolescence.
* The market value of other properties in the area in which the property is located.
* The value of net income received from the property. Rentals are often capitalized and then adjusted for depreciation. The same principle can be applied to gross rents. This method, however, must be adjusted to account for operating costs.
* The value accepted by state probate courts for purposes of state death taxes, if based on appraisals made by qualified real estate experts.
* Prices at which comparable property in the same area was sold at a time near the applicable valuation date, provided it was an arm’s length transaction for the best price obtainable. Usually, more than one comparable property sale will be used, especially where the property to be valued is a personal residence or undeveloped acreage.
* Cost to duplicate the property, taking depreciation into account. The cost or value of land would have to be separated from the total value. The cost of reproducing the building, using present cost figures, would have to be estimated, and then the loss in value due to depreciation would have to be subtracted from the total of the other two figures.
* Unusual facts.

74
Q

What would value be in the event of a sale of real property within a reasonable period of time after the decedent’s death?

Land?

A

In the event of a sale of real property within a reasonable period of time after the decedent’s death, in such a manner as to ensure the highest possible price, the amount received will usually be accepted as its value. Unaccepted offers to purchase the property will also be considered. The price of a sale at auction will be accepted only if it appears that there was no other method that would have obtained a higher price.

Land does not have to produce income or have an active market to attain substantial value. Where lands are in or adjacent to a settled community, owners frequently hold such lands in anticipation of realizing their true value from future sales. For example, a home at the edge of an expanding shopping center might be worth far more to the shopping center developer than it would be to a potential buyer in the residential market.

75
Q

What is Special Use Valuation (Section 2032A)?

A

An executor may elect to value qualifying real property on the basis of its actual special use rather than its highest and best use.
* This rule is especially useful where the price of farmland is artificially increased by, or has not kept up with, the price per acre of encroaching housing developments or more lucrative commercial businesses.
* Electing a valuation based on special use enables the executor to value the farmland for farming purposes by applying the inflation-adjusted reduction amount.

The maximum reduction of the decedent’s gross estate under this provision in 2023 is $1,310,000.

76
Q

What are the Qualification Requirements for Special Use Valuation (Section 2032A)?

A

The following are the requirements for a property to qualify for the special valuation rule:
* On the date of the decedent’s death, the property must be involved in a qualified use. The term qualified use is defined by IRC Section 2032A(b)(2) as use of a farm for farming purposes or in a trade or business other than farming.
* The value of the qualified property in the decedent’s estate must equal at least 50% of the decedent’s gross estate. This is after debts or unpaid mortgages are deducted from both the qualified property as well as the gross estate.
* At least 25% of the gross estate, less debts and unpaid mortgages on all property in the gross estate, must be qualified farm or closely held business real property.
* All gifts made within three years of the donor’s death are added back into the gross estate solely for the purpose of calculating the 50% and 25% tests.
* Such property must pass to a qualified heir. The term qualified heir is defined at IRC Section 2032A(e) to include the decedent’s:
* Immediate family,
* Ancestors,
* Lineal descendants,
* Spouse or the spouse of a descendant, or
* A grandparent’s lineal descendant.

  • The real property must have been owned by the decedent or a member of his or her family and used as a farm or in a closely held business for an aggregate of five years or more of the eight-year period ending on the date of the decedent’s death.
  • During this period the decedent or a member of his or her family must have been a material participant in the operation of the farm or other business.
77
Q

Describe Section IRC 2032(A)

A

If a farm or a closely held business qualified for the special valuation rules and its value so determined is used for federal estate tax purposes, then an additional estate tax will be imposed if within 10 years after the decedent’s death and before the death of the qualified heir, the qualified heir who receives such property disposes of any interest in that property, other than to a qualified family member, or ceases to use that property in the manner in which it was used to qualify for this special tax treatment.

Generally, the additional tax imposed shall be the excess of the tax that would have been imposed on the property if it were valued at its best use over the tax imposed because the property was valued at its qualified use value.

78
Q

Describe the Valuation of Life Insurance

A

Proceeds of life insurance on the life of the decedent receivable by or for the benefit of the estate will be included in the insured decedent’s estate and will also be subject to probate.
In addition, where the decedent held incidents of ownership in a policy, such ownership will invoke inclusion in the decedent’s estate.
The amount includable is the amount receivable by the beneficiary. This includes dividends and premium refunds. In determining how much is includable, no distinction is made between an ordinary life policy, a term policy, group insurance, or an accidental death benefit.

If a settlement option is elected, the amount that would have been payable as a lump sum is the amount includable. If the policy did not provide for a lump sum payment, the amount includable is the commuted amount used by the insurance company to compute the settlement option payments.

The value of policy where the insured has not died is included in the policy owner’s gross estate where he or she predeceases the insured:
* If a new policy is involved, its value will be the gross premium paid.
* If the policy is paid-up or a single premium policy, its value is the replacement cost. This is the single premium which that company would have charged for a comparable contract of equal face value on the life of a person who was the insured’s age at the time the decedent-policyholder died.
* If the policy is an established whole life policy, its value is found by adding any unearned portion of the last premium to the interpolated terminal reserve.
* If the policy is a term policy, its value is the unused premium.

79
Q

Describe the Valuation of Annuities

A

Annuities can be commercial or noncommercial. Where the annuity is noncommercial, such as a private annuity, the present value of future payments determines its fair market value.
* Likewise, for gift and estate tax purposes, the fair market value of life estates, terms for years, remainders, and reversions is their present value.

Annuities, life estates, reversions, remainders, and terms for years are valued according to a discount rate that changes monthly. They may be based on a life or on the joint lives of two or more people.

There are two steps involved in this process:
* Convert the length of the lifetime involved, to a life expectancy. This is done by means of a mortality table based on a study of the longevity of a large number of people over a selected period of time, which indicates the expectancy of life in years for an individual who represents the average experience at each age of life.
* **Determine a proper discount interest rate
. When that interest rate is determined, a valuation table can be constructed that converts the two factors to be applied, life expectancy and the rate of interest, into one factor. This f
actor is applied to the amount of the periodic payment in order to determine the present worth or value of the property interest**.
* Standard valuation tables are used to conduct term and life computations. The valuation must be based on an interest rate that is rounded to the nearest 2/10ths of one percent. The interest rate is equal to 120% of the federal midterm rate in effect for the month in which the valuation date falls.

80
Q

What are the 3 qualifications that need to be added to this explanation of the law respecting valuation?

A

Two qualifications need to be added to this explanation of the law respecting valuation:
* These valuation provisions do not apply to interests valued with respect to qualified plans including tax-sheltered annuities and IRAs or in other situations specified in Treasury regulations.
* If an income, estate, or gift tax charitable contribution is allowable for any part of the property transferred, the taxpayer may elect to use the discount rate for either of the two months preceding the month in which the valuation date falls. However, if a transfer of more than one interest in the same property is made with respect to which the taxpayer could use the same interest rate, such interest rate is to be used with respect to each such interest.

81
Q

Describe Commercial Annuities

A

Commercial annuities are annuities under contracts issued by companies regularly engaged in their sale.
* They are valued by reference to the price at which the company issues comparable contracts.
* A retirement income policy, from the point in time, that there is no longer an insurance element, is treated as a contract for the payment of an annuity.

82
Q

Describe the Valuation of Listed Stocks

A

Where a stock has an established market and quotations are available to value the stock as of the date in question, the fair market value per share on the applicable valuation date governs the valuation for both gift and estate tax purposes.

The FMV is based on selling prices when there is a market for the stock or bond. This would be the mean between the highest and lowest quoted selling price on the valuation date. If there were no sales on the valuation date, but there were sales on dates within a reasonable period both before and after the valuation date, the FMV is determined by taking a weighted average of the means between the highest and lowest sales on the nearest date before and the nearest date after the valuation. The average is then weighted inversely by the respective number of trading days between the selling date and the valuation date.

Where there is a large block of stock that could not be marketed in an orderly manner, the block might depress the market because it could not be converted to cash as readily as could a few shares. Therefore, selling prices and bid and asked prices may not reflect fair market value.
* Sometimes it may be necessary to value this type of stock as if it were closely held and not actively traded. If this can be established, a reasonable modification of the normal basis for determining fair market value can be made.

As discussed in the previous Module, a blockage discount is determined by the effect that the block would have had on the market if it were sold over a reasonable period of time and in a prudent manner.
* A similar situation occurs where sales at or near the date of death are few or sales are of a sporadic nature and may not indicate a FMV.

The opposite of the blockage situation above is where the block of stock to be valued represents a controlling interest either actual or effective in a going business. Here, the price of normally traded shares may have little relation to the true value of the controlling lot. The large block could have the effect of increasing value because of its element of control.

83
Q

Section 3 - Valuation Summary

For estate-tax purposes, property is valued at its fair market value at the time of a person’s death, unless the alternate valuation date, which is six months later, is elected. Fair market value is defined as the price at which property would change hands between a willing buyer and a willing seller, both with reasonable knowledge of relevant facts.

In this lesson, we have covered the following:
* Valuation: Defined as the technique used to value property or business interests for tax purposes. Transferring interests in property to other family members, either outright or in trust, is a method of reducing the value of interest retained at death. Election of an alternative valuation date also results in decreasing the aggregate value of the gross estate during the six-month period following the date of death. The appraiser’s objective while valuing the gross estate must be to derive fair and sound value, which if litigated, must be sustained by the court.
* Valuation of real property: In the absence of a market for such property follows the rule that the highest price available or the salvage value, whichever is greater, will control the valuation. If there is a market for such property, several factors such as nature, condition, size, location, use, suitability, zoning restrictions, income from the property, depreciation, and prices of comparable property will affect the valuation. In certain circumstances, an executor may elect to value qualifying real property on the basis of its actual special or business use value rather than its fair market value at its highest and best use.

A
  • Valuation of life insurance: This process includes proceeds, dividends, and premium refunds from all types of policies such as ordinary life policy, a term policy, group insurance, or accidental death benefits. The amount payable as a lump sum or the commuted amount used by the insurance company to compute the settlement is the amount includable in the gross estate.
  • Valuation of annuities: Private annuity valuation starts with the step of converting the length of a lifetime involved into life expectancy using a mortality table. Next, the proper discount rate is established.** Valuation tables based on these two steps are used to do term and life computations.** Commercial annuities are valued by reference to the price at which the company issues comparable contracts.
84
Q

For federal estate tax purposes, an unmatured life insurance policy owned by the decedent on the life of another is valued as follows:
* An unmatured policy owned by the decedent on the life of another is not includable in the decedent’s estate
* If the policy is paid up, the policy is valued at its replacement cost
* If the policy is a new policy, the policy is valued at its interpolated terminal reserve plus any unearned premium
* If the policy is a term policy, the policy is valued at the lower of cost or standard valuation tables

A

If the policy is paid up, the policy is valued at its replacement cost
* The value of an unmatured policy owned by a decedent on the life of another is includable in the gross estate where he or she predeceases the insured. If the policy is paid up, its value is its replacement cost. If the policy is new, the value is the gross premium paid. If the policy is a term policy, the value is the unused premium.

85
Q

Cedric left a huge estate to his daughter, Lucy, when he died. The executor of his estate approaches you to advise him about the valuation of Cedric’s estate. What do you think could be the various factors that will affect the valuation of Cedric’s real property? (Select all that apply)
* The value of net income received from the property
* The size, shape and location of the property
* The nature and condition of the property
* An active market for the land
* The actual and potential use of the property

A

The value of net income received from the property
The size, shape and location of the property
The nature and condition of the property
The actual and potential use of the property
* Some of the basic factors that affect valuation of real property are the size, shape and location, the nature and condition, the actual and potential use of the property, the value of net income received, and zoning restriction.
* However, land does not have to have an active market to attain substantial value.

86
Q

A blockage discount occurs when: (Select all that apply)
* The block of stock to be sold is too large to be marketed in an orderly manner
* Selling prices and bid and asked prices may not reflect the fair market value
* The block of stock valued represents a controlling interest in the going business
* The block of stock has the effect of increasing value because of its element of control

A

The block of stock to be sold is too large to be marketed in an orderly manner
Selling prices and bid and asked prices may not reflect the fair market value
* A blockage discount occurs when a large block of stock that cannot be marketed in an orderly manner results in depressing the market. If this is the case, selling prices and bid and asked prices may not reflect their fair market value. The converse of the blockage situation is where the block of stock to be valued represents a controlling interest either actual or effective in a going business. Here, the large block could have the effect of increasing value because of its element of control.

87
Q

Module Summary

The federal estate tax is a tax on a person’s right to transfer property at death. The first step in calculating the federal estate tax is to measure the value of the decedent’s gross estate. The gross estate is the value of all property or interests in property owned or controlled by the deceased. The value for estate tax purposes is the fair market value of the property at the date of death or, if a lower estate value would result, six months after death, which is called the alternate valuation date (AVD).

The key concepts to remember are:
* Gross Estate: The gross estate is composed of property that the decedent owned outright at the time of his or her death, as well as those in which he or she had a beneficial interest, or a dower and curtesy interest. Generally, property transferred as gifts within the three years before the death of the decedent is not included in the gross estate. However, certain assets gifted within three years of death are includable. For example, the decedent retained life estate in the gifted property. Additionally, the value of gifts taking effect only upon the death of the donor is also included because of the likelihood of the gift reverting back to the donor. These are properties in which the decedent had a reversionary interest.

A
  • Other IRC Sections: Gifts wherein the decedent retained the right to alter, amend, revoke, or terminate the gift are revocable transfers and their value is included in the gross estate. To the extent that the decedent paid for the contract, annuity payments that are receivable by a beneficiary after the death of a decedent are includable in the gross estate. The 50-50 rule for property jointly-owned by husband and wife and the percentage-of-contribution rule for others are used to determine the value of jointly-owned property included in the gross estate. Property over which the decedent held a general power of appointment is also included in the gross estate unless limited by an ascertainable standard. If the decedent retained incidents of ownership in life insurance policies, these are also included. If the decedent spouse’s estate elected to receive the marital deduction for qualifying income interest property, the remaining value of these assets must be included in the estate of the surviving spouse.
  • Valuation: To calculate the value of the gross estate, the property is valued at fair market value. This is the price at which property would change hands between a willing buyer and willing seller. Real property is valued at the highest price available if a market for the property is not available. If a market is available, several factors related to the nature of the property, zoning restrictions, and prices of comparable property must be considered. The lump-sum amount or the commuted amount is includable in the gross estate. Private annuities are valued based on government valuation tables while the value of commercial annuities is determined with reference to the price charged by the company for similar contracts.
88
Q

Where a stock has an established market and quotations are available for the date in question, the fair market value per share governs the valuation for
I. gift tax purposes.
II. estate tax purposes.
* II only
* I only
* Neither I nor II
* Both I and II

A

Both I and II
* Where a stock has an established market and quotations are available to value the stock as of the date in question, the fair market value per share on the applicable valuation date governs the valuation for both gift and estate tax purposes.

89
Q

According to Section 2034, the gross estate is:
I. reduced by the value of a dower.
II. reduced by the value of a curtesy.
III. not reduced by the value of a dower.
IV. not reduced by the value of a curtesy.
* II and III
* I and II
* I and IV
* III and IV
According to Section 2034, the gross estate is not reduced by the value of any dower or curtesy interest.
The value of these assets will be included in the decedent’s gross estate.

A

III and IV
* According to Section 2034, the gross estate is not reduced by the value of any dower or curtesy interest.
The value of these assets will be included in the decedent’s gross estate.

90
Q

The maximum reduction of the decedent’s gross estate under the special use valuation in 2023 is __ ____??____ __.
* $1,275,000
* $1,500,000
* $1,230,000
* $1,310,000

A

$1,310,000
* The maximum reduction of the decedent’s gross estate under Section 2032(A) (special use valuation) in 2023 is $1,310,000.
* This rule is beneficial when farmland prices fall behind local prices for new homes or commercial property.

91
Q

A decedent retained the right to designate who would use, possess, and enjoy a property.
For the value of the property to be included in the decedent’s gross estate, the right to designate must have been retained by the decedent for:
I. life.
II. a period not ascertainable without reference to death.
III. a period that ends before death.
IV. a period that does not end before the decedent’s death.
* I and II
* I only
* III only
* I, II, and IV

A

I, II, and IV

  • For an asset to be included in the gross estate under 2036, the right must have been retained by the decedent for:
  • Life
  • Period not ascertainable without reference to death
  • Period that does not end before the decedent’s death
92
Q

Property over which a general power of appointment is held will be includable in a decedent’s gross estate in which of these situations?
I. The general power could be exercised at death.
II. The general power of appointment could be exercised only during lifetime.
* I only
* Neither I nor II
* II only
* Both I and II

A

Both I and II
* A general power of appointment allows the decedent to transfer the property to himself, his estate, his creditors, or the creditors of his estate.
Such property will be includable in a decedent’s gross estate regardless of whether the power could be exercised at death or only during lifetime
.

93
Q

Identify the term that refers to the widow’s property rights under state law.
* Curtesy
* Bestowal this.
* Endowment
* Dower

A

Dower
* Under the common law system, a surviving spouse has dower or curtesy rights.
* Dower is defined as the widow’s property rights under state law.
* Curtesy is defined as the widower’s property rights under state law.

94
Q

Select the item that would NOT be considered future income rights for income in respect of a decedent (IRD).
* royalties
* rents
* dividends
* paid salary

A

paid salary
* Unpaid salary is considered a future income right.

95
Q

Carl and his wife, Louise, purchase a beach house for $200,000, and hold the property as tenancy by the entirety. Twenty years later, when the beach house is valued at $500,000, Louise passes away.
If the entire contribution were made from Louise’s salary, what amount would be included in her gross estate?
* $200,000
* $500,000
* $100,000
* $250,000

A

$250,000
* Only 50% of the value of the beach house, or $250,000 ($500,000 × 0.50), will be included in Louise’s gross estate due to the tenancy by the entirety titling.

96
Q

Stan and his wife, Shelley, purchase 100 shares of BIFX stock for $50,000, and hold the property as joint tenants with rights of survivorship.
If the entire contribution were made from Stan’s salary, what percentage would be included in his gross estate?
* 75%
* 25%
* 100%
* 50%

A

50%
* Only 50% will be included in Stan’s gross estate due to the JTWROS titling.

97
Q

In the absence of a market for real property, the greater of the following will control the valuation:
I. The highest price available
II. The salvage value
II only
I only
Both I and II
Neither I nor II

A

Both I and II

  • In the absence of a market for such property, the greater of the following will control the valuation:
  • The highest price available, or
  • The salvage value.
98
Q

Corporate owned life insurance (COLI) is includable in a decedent’s estate if he or she owned more than __ ____??____ __ of the corporation’s stock at death and is the insured on the policy.
* 75%
* 50%
* 80%
* 60%

A

50%

  • Corporate owned life insurance (COLI) is includable in a decedent’s estate if he or she owned more than 50% of the corporation’s stock at death and is the insured on the policy, but only to the extent that it is payable to a party other than the corporation or its creditors.
99
Q

Select the item that would NOT be considered future income rights for income in respect of a decedent (IRD).
* dividends
* interest payments
* vested amounts in qualified retirement plans
* business accounts payable

A

business accounts payable
* Business accounts receivable are considered future income rights.

100
Q

The alternate valuation date (AVD) is ________ after death.
* six months
* twelve months
* three months
* nine months

A

six months

  • The alternate valuation date (AVD) is six months after death.
  • The executor can elect the alternate valuation date only if the value of the decedent’s gross estate has diminished in value in six months and the estate tax liability is also less.
101
Q

Under the special use valuation provisions in Section 2032(A), if a farm or a closely held business qualified for the special valuation, an additional estate tax will be imposed if a qualified heir sells the property within __ ____??____ __ after the decedent’s death.
* 5 years
* 3 years
* 10 years
* 8 years

A

10 years

  • If a farm or a closely held business qualified for the special valuation rules, then an additional estate tax will be imposed if within 10 years after the decedent’s death a qualified heir disposes of the property or ceases to use that property in the manner in which it was used to qualify for this special tax treatment.
102
Q

Which type(s) of power(s) of appointment is the ascertainable standard allowing trust income and principal to be used for the holder’s health, education, maintenance, and support (HEMS)?
I. General Power of Appointment
II. Special Power of Appointment
* Neither I nor II
* II only
* I only
* Both I and II

A

II only

  • Holding control over trust income and corpus for health, education, maintenance, and support (HEMS) is a special power of appointment.
  • If the trust document says that the holder can use trust property for health, education, and “comfort,” or any wording other than HEMS, the power of appointment will be considered a general power instead of a special power.
103
Q

Select the transaction that would be used to determine value according to IRS valuation standards.
* A disposition in which one of the parties was ignorant of a material fact.
* A sale outside of regular business channels.
* A forced sale price.
* The price at which a comparable item would have been sold at retail.

A

The price at which a comparable item would have been sold at retail.
* Generally, the IRS position is that the price at which the item or a comparable item would have been sold at retail is determinative.

104
Q

Choose the scenario(s) that may present valuation problems.
I. A property with both wholesale and retail markets.
II. An appraisal of the worth must be made based on comparison to similar property.
III. The property in question is unique, such as a patent or copyright.
IV. There is an established market for identical property.
* III and IV
* IV only
* I and III
* I, II, and III

A

I, II, and III
Valuation problems become essentially problems of evidentiary proof, and often of opinion, where:
* There are different markets for the same property.
* The appraisal of the worth must be made on the basis of comparison with somewhat similar property.
* The property in question is unique, such as a patent or copyright

105
Q

Each of the following are examples of property whose value will be included in the decedent’s gross estate under IRC Section 2033 except:
* Property in which the decedent held a sufficient interest
* Property in which the decedent held a vested remainder interest
* Property in which the decedent held a contingent remainder interestY
* Specific property

A

Property in which the decedent held a contingent remainder interestY

Examples of property whose value will be included under IRC Section 2033 include:
* Specific property (e.g., stocks, bonds)
* Property in which the decedent held a sufficient interest
* Property in which the decedent held a vested remainder interest

Property in which the decedent held a contingent remainder interest is not considered includible in the decedent’s gross estate under IRC Section 2033.

106
Q

According to the general valuation rules, assets included in the gross estate are valued on the
* decedent’s date of death.
* date of transfer to the beneficiary.
* date Form 706 is received.
* original purchase date.

A

decedent’s date of death.

  • According to the general rules, assets included in the gross estate are valued on the decedent’s date of death.
107
Q

Identify the term that refers to the widower’s property rights under state law.
* Endowment
* Bestowal
* Dower
* Curtesy

A

Curtesy
* Under the common law system, a surviving spouse has dower or curtesy rights.
* Dower is defined as the widow’s property rights under state law.
* Curtesy is defined as the widower’s property rights under state law.

108
Q

Each of the following gifts made by the decedent will be included back in the gross estate except
* creating a revocable trust and transferring property to the trust.
* creating a life estate and gifting the remainder interest.
* keeping a reversionary interest in the property gifted away.
* making a revocable trust irrevocable and living for at least three-years.

A

making a revocable trust irrevocable and living for at least three-years.
* The donor or grantor can relinquish property interests by giving up the life estate or the reversionary interest, or by making the revocable trust irrevocable. This will avoid inclusion in the grantor’s gross estate if he outlives that transfer for more than three years.

109
Q

Identify the qualification requirements for special use valuation under Section 2032(A).
I. The qualified property in the decedent’s estate must equal at least 50% of the decedent’s gross estate.
II. At least, 25% of the gross estate must be qualified farm or closely held business real property.
III. Property must pass to a qualified heir.
IV. On the date of the decedent’s death, the property must be involved in a qualified use.
* I and II
* IV only
* I, III, and IV
* I, II, III, and IV

A

I, II, III, and IV

110
Q

Property over which a general power of appointment is held will be includable in a decedent’s gross estate in which of these situations?
I. The general power could be exercised at death.
II. The general power of appointment could be exercised only during lifetime.
* II only
* I only
* Neither I nor II
* Both I and II

A

Both I and II
* A general power of appointment allows the decedent to transfer the property to himself, his estate, his creditors, or the creditors of his estate.
* Such property will be includable in a decedent’s gross estate regardless of whether the power could be exercised at death or only during lifetime.

111
Q

The basic factors that affect valuation of real property, where there is a market for the property include each of the following except.
* Suitability of the property is for its actual or intended use.
* Cost to duplicate the property, taking depreciation into account.
* The price of a sale at auction.
* Zoning restrictions.

A

The price of a sale at auction.

  • The price of a sale at auction, will be accepted only if it appears that there was no other method that would have obtained a higher price.
112
Q

Elizabeth and her daughter, Lisa, purchased 100 shares of stock for $90,000. Elizabeth contributed $60,000.
At Elizabeth’s death, the stock has a value of $120,000. What percentage of the FMV on the date of death would be includable in Elizabeth’s estate?
* 100%
* 67%
* 50%
* 33%

A

67%

  • The percentage-of-contribution rule applies to the estate taxation of property held jointly with the right of survivorship where the joint owners are not spouses, or if one of the spouses is not a U.S. citizen.
  • The estate tax includability of jointly-held property with survivorship rights involving a mother and daughter is the portion of the purchase price attributable to the decedent’s contribution.
  • Since Elizabeth contributed 60% of the original purchase ($60,000 ÷ $90,000 = 0.67), 67% of the value at death will be included in Elizabeth’s estate ($120,000 × 0.67 = $80,000).
113
Q

McCoy and his son, Elvin, purchased 200 shares of stock for $100,000. McCoy contributed $60,000.
At McCoy’s death, the stock has a value of $120,000. How much would be includable in McCoy’s estate?
* $100,000
* $60,000
* $120,000
* $72,000

A

$72,000
* The percentage-of-contribution rule applies to the estate taxation of property held jointly with the right of survivorship where the joint owners are not spouses, or if one of the spouses is not a U.S. citizen.
* The estate tax includability of jointly-held property with survivorship rights involving a father and son is the portion of the purchase price attributable to the decedent’s contribution.
* Since McCoy contributed 60% of the original purchase ($60,000 ÷ $100,000 = 0.60), 60% of the value at death will be included in McCoy’s estate ($120,000 × 0.60 = $72,000).

114
Q

Under the IRC Regulations, fair market value (FMV)
* all of these statements are correct.
* involves a transaction where neither the buyer or seller is under any compulsion to buy or to sell.
* requires that the willing buyer and willing seller both have reasonable knowledge of relevant facts involved in a potential sale..
* is the price at which the property would change hands between a willing buyer and a willing seller.

A

all of these statements are correct.

  • Under the IRC Regulations, fair market value (FMV) is “the price at which the property would change hands between a willing buyer and a willing seller, neither being under any compulsion to buy or to sell and both having reasonable knowledge of relevant facts.”